Administración     

Olimpiadas de Matemáticas
Página de preparación y problemas

OME Local
OME Nacional
OIM
OME Andalucía
Retos UJA
Selector
La base de datos contiene 1154 problemas y 775 soluciones.

XIX Olimpiada Iberoamericana de Matemáticas — 2004

Sesión 1 —  Castellón (España), 21 de septiembre de 2004

Problema 516
Un conjunto $S$ de enteros positivos se llama canalero si para cualesquiera tres números $a, b, c\in S$, todos diferentes, se cumple que $a$ divide a $bc$, $b$ divide a $ca$ y $c$ divide a $ab$.
  1. Demostrar que, para cualquier conjunto finito de enteros positivos $\{c_1,c_2,\ldots,c_n\}$, existen infinitos enteros positivos $k$ tales que el conjunto $\{kc_1,kc_2,\ldots, kc_n\}$. es canalero.
  2. Demostrar que, para cualquier entero $n\geq 3$, existe un conjunto canalero que tiene exactamente $n$ elementos y ningún entero mayor que $1$ divide a todos sus elementos.
Sin pistas
Sin soluciones
info
Si crees que el enunciado contiene un error o imprecisión o bien crees que la información sobre la procedencia del problema es incorrecta, puedes notificarlo usando los siguientes botones:
Informar de error en enunciado Informar de procedencia del problema
Problema 517
Sean $X$ e $Y$ los extremos de un diámetro de una circunferencia $\Gamma$ y sea $N$ el punto medio de uno de los arcos $XY$ de $\Gamma$. Sean $A$ y $B$ dos puntos en el segmento $XY$. Las rectas $NA$ y $NB$ cortan nuevamente a $\Gamma$ en los puntos $C$ y $D$, respectivamente. Las tangentes a $\Gamma$ en $C$ y $D$ se cortan en $P$. Sea $M$ el punto de intersección del segmento $XY$ con el segmento $NP$. Demostrar que $M$ es el punto medio del segmento $AB$.
Sin pistas
Sin soluciones
info
Si crees que el enunciado contiene un error o imprecisión o bien crees que la información sobre la procedencia del problema es incorrecta, puedes notificarlo usando los siguientes botones:
Informar de error en enunciado Informar de procedencia del problema
Problema 518
Sea $A = \{1,2,3,...,n\}$ con $n\gt 5$. Demostrar que existe un conjunto finito $B$ de enteros positivos distintos que contiene a $A$ y tal que el producto de los elementos de $B$ es igual a la suma de los cuadrados de los elementos de $B$.
Sin pistas
Sin soluciones
info
Si crees que el enunciado contiene un error o imprecisión o bien crees que la información sobre la procedencia del problema es incorrecta, puedes notificarlo usando los siguientes botones:
Informar de error en enunciado Informar de procedencia del problema

Sesión 2 —  Castellón (España), 22 de septiembre de 2004

Problema 519
Sean $\Gamma$ una circunferencia de centro $O$, $AE$ un diámetro de $\Gamma$ y $B$ el punto medio de uno de los arcos $AE$ de $\Gamma$. El punto $D\neq E$ está sobre el segmento $OE$. El punto $C$ es tal que el cuadrilátero $ABCD$ es un paralelogramo con $AB$ paralelo a $CD$ y $BC$ paralelo a $AD$. Las rectas $EB$ y $CD$ se cortan en el punto $F$. La recta $OF$ corta al arco menor $EB$ de $\Gamma$ en el punto $I$. Demostrar que la recta $EI$ es la bisectriz del ángulo $BEC$.
Sin pistas
Sin soluciones
info
Si crees que el enunciado contiene un error o imprecisión o bien crees que la información sobre la procedencia del problema es incorrecta, puedes notificarlo usando los siguientes botones:
Informar de error en enunciado Informar de procedencia del problema
Problema 520
Sean $A$ y $B$ dos conjuntos tales que:
  • $A\cup B$ es el conjunto de los enteros positivos;
  • $A\cap B$ es el vacío;
  • si dos enteros positivos tienen como diferencia a un primo mayor que $2013$, entonces uno de ellos está en $A$ y el otro en $B$.
Hallar todas las posibilidades para los conjuntos $A$ y $B$.
Sin pistas
Sin soluciones
info
Si crees que el enunciado contiene un error o imprecisión o bien crees que la información sobre la procedencia del problema es incorrecta, puedes notificarlo usando los siguientes botones:
Informar de error en enunciado Informar de procedencia del problema
Problema 521
Una configuración es un conjunto finito $S$ de puntos del plano entre los cuales no hay tres colineales y a cada punto se le asigna algún color, de modo que si un triángulo cuyos vértices están en $S$ tiene un ángulo mayor o igual a $120º$, entonces exactamente dos de sus vértices son de un mismo color. Hallar el número máximo de puntos que puede tener un configuración.
Sin pistas
Sin soluciones
info
Si crees que el enunciado contiene un error o imprecisión o bien crees que la información sobre la procedencia del problema es incorrecta, puedes notificarlo usando los siguientes botones:
Informar de error en enunciado Informar de procedencia del problema
José Miguel Manzano © 2010-2024. Esta página ha sido creada mediante software libre